0 Daumen
161 Aufrufe

Es seien \( a_{1}, \ldots, a_{k} \) natürliche Zahlen, so dass die Zahl \( a_{1} \cdot \ldots \cdot a_{k}+1 \) durch 3 teilbar ist.
(a) Zeigen Sie, dass keine der Zahlen \( a_{1}, \ldots, a_{k} \) durch 3 teilbar ist.
(b) Zeigen Sie, dass mindestens eine der Zahlen \( a_{1}+1, \ldots, a_{k}+1 \) durch 3 teilbar ist.

Avatar von

1 Antwort

0 Daumen

(a)

Nimm an, dass eine der Zahlen durch 3 teilbar ist. Zeige dann, dass es einen Widerspruch gibt.

(b)

ai+1 durch 3 teilbar bedeutet ai=3m+2.

Da kein ai wegen (a) durch 3 teilbar ist, betrachten wir den Fall, dass alle ai die Form 3m+1 haben, d.h. ai≡1 mod 3.

Dann gilt aber

 \( a_{1} \cdot \ldots \cdot a_{k}+1 \) ≡2 mod 3, also ein Widerspruch zur Voraussetzung.

Avatar von 47 k

Ein anderes Problem?

Stell deine Frage

Willkommen bei der Mathelounge! Stell deine Frage einfach und kostenlos

x
Made by a lovely community